2018年考研试题 - Eufisky - The lost book
醉鬼能回家,但喝醉的鸟儿可能永远回不了家!
学界与工业界的AI研究:有哪些重要不同?

2018年考研试题

Eufisky posted @ 2017年12月26日 03:10 in 机器学习 with tags 考研 , 850 阅读
证明积分不等式:
 
\[\frac{1}{5}<\int_0^1\frac{xe^xdx}{\sqrt{x^2-x+25}}<\frac{2}{\sqrt{99}}.\]
 
证明  注意到
\[x^2-x+25=\left(x-\frac{1}{2}\right)^2+\frac{99}{4}>\frac{99}{4}, a.e. x\in [0,1],\]
从而
\[\int_0^1\frac{xe^xdx}{\sqrt{x^2-x+25}}<\frac{2}{\sqrt{99}}\int_0^1xe^xdx=\frac{2}{\sqrt{99}}.\]
另一方面, 分部积分, 得到
\[\int_0^1\frac{xe^xdx}{\sqrt{x^2-x+25}}=\frac{(x-1)e^x}{\sqrt{x^2-x+25}}\Big|_0^1+\int_0^1\frac{(x-1)(x-\frac{1}{2})e^x}{\sqrt{(x^2-x+25)^3}}dx=\frac{1}{5}+\int_0^1\frac{(x-1)(x-\frac{1}{2})e^x}{\sqrt{(x^2-x+25)^3}}dx.\]
\[f(x)=\frac{x-\frac{1}{2}}{\sqrt{(x^2-x+25)^3}},g(x)=(x-1)e^x,\]
\[f'(x)=\frac{-2x^2+2x+\frac{97}{4}}{\sqrt{(x^2-x+25)^5}}>0,\forall x\in [0,1],\int_0^1 f(x)dx=0, g'(x)=xe^x,\]
因此$f,g$在$[0,1]$上严格递增. 根据Chebyshev 积分不等式,
\[\int_0^1\frac{(x-1)(x-\frac{1}{2})e^x}{\sqrt{(x^2-x+25)^3}}dx=\int_0^1f(x)g(x)dx>\int_0^1 f(x)dx\int_0^1g(x)dx=0.\]
\[\int_0^1\frac{xe^xdx}{\sqrt{x^2-x+25}}>\frac{1}{5}.\]
最后得到
\[\frac{1}{5}<\int_0^1\frac{xe^xdx}{\sqrt{x^2-x+25}}<\frac{2}{\sqrt{99}}.\]

2018年武汉大学653数学分析
 
一(30分).1.计算极限$$\lim_{n\rightarrow\infty}\sum_{k=n^2}^{(n+1)^2}\frac1{\sqrt k}.$$
    2.计算极限$$\lim_{n\rightarrow\infty}\frac{\int_0^\mathrm\pi\sin^nx\cos^6x\operatorname dx}{\int_0^\mathrm\pi\sin^nx\operatorname dx}$$
    3.已知$x_{n+1}=\ln\left(1+x_n\right)$,且$x_1>0$,计算$$\lim_{n\rightarrow\infty}nx_n$$
 
二.设$f(x),f_1(x)$在$[a,b]$区间上连续,$f_{n+1}(x)=f(x)+\int_a^x\sin\{f_n(t)\}\operatorname dt$,证明:$\{f_n\}$在$[a,b]$一致收敛.
 
 
三.设$$f(x)=\left\{\begin{array}{lc}e^{-\frac1{x^2}}&,\;x\neq0\\0&,\;x=0\end{array}\right.$$证明$f(x)$在$x=0$处任意阶导数存在.
 
 
四.已知$(x_1,x_2,x_3)\in{R}^3$,其中$u=\frac1{\left|x\right|},\left|x\right|=\sqrt{x_1^2+x_2^2+x_3^2}$,计算
$$\oint\limits_S\frac{\partial^2 u}{\partial x_i\partial x_j}{\rm d}S,i,j=1,2,3$$,其中$S:x_1^2+x_2^2+x_3^2=R^2$
 
五.讨论求解方程$f(x)$牛顿切线法.1.推导牛顿切线法迭代公式;
                                                   2.在适当的条件下,证明牛顿切线法收敛
 
六(20分).求极限$$\lim_{n\rightarrow\infty}(nA-\sum_{k=1}^nf(\frac kn))=B$$存在时,$A,B$的值。
 
七.设$u_i=u_i(x_1,x_2),i=1,2$,且关于每个变量为周期1的连续可微函数,求$$\iint\limits_{0\leq x_1,x_2\leq1}det(\delta_{ij}+\frac{\partial u_i}{\partial x_j})dx_1dx_2,$$其中$det(\delta_{ij}+\frac{\partial u_i}{\partial x_j})$是映射$x\mapsto(x_1+u_1,x_2+u_2)$的雅克比行列式.
 
八(40分).设$f(x)$在$[a,b]$上$Riemann$可积,$\varphi(x)$是周期为$T$的连续函数,证明:
        1.存在阶梯函数$g_\varepsilon(x)$使得$$\int_a^b\left|f(x)-g_\varepsilon(x)\right|\operatorname dx<\frac\varepsilon2$$
        2.计算$$\lim_{n\rightarrow\infty}\int_a^b\varphi(nx)\operatorname dx$$
        3.证明$$\lim_{n\rightarrow\infty}\int_a^bf(x)\varphi(nx)\operatorname dx=\frac1T\int_0^T\varphi(x)\operatorname dx\int_a^bf(x)\operatorname dx$$
        4.计算$$\lim_{n\rightarrow\infty}\frac1{\ln n}\int_0^T\frac{\varphi(nx)}xdx,其中函数\frac{\varphi(nx)}x收敛$$

试题(1b):证明\[\lim_{n\to +\infty}\left(\int_0^1\frac{\sin x^n}{x^n}dx\right)^n=\prod_{k=1}^{+\infty}\exp{\left(\frac{(-1)^k}{2k(2k+1)!}\right)}\]
 
作代换$x^n=y$
\[\int_0^1\frac{\sin x^n}{x^n}dx=\frac{1}{n}\int_0^1\frac{\sin y}{y^{2-\frac{1}{n}}}dy\]
\[\int_0^1 y^{\frac{1}{n}-2}\sum_{k=0}^{+\infty}\frac{(-1)^k}{(2k+1)!}y^{2k+1}dy=\int_0^1 y^{\frac{1}{n}-1}dy+\int_0^1\sum_{k=1}^{+\infty}\frac{(-1)^k}{(2k+1)!}y^{2k-1+\frac{1}{n}}dy\]
容易得到
\[\int_0^1 y^{\frac{1}{n}-1}dy=n,\left|\frac{(-1)^k}{(2k+1)!}y^{2k-1+\frac{1}{n}}\right|\le \frac{1}{(2k+1)!}\]
由优级数判别法后一项中的函数项级数一致收敛,可以逐项积分,上式
\[=n+\sum_{k=1}^{+\infty}\frac{(-1)^k}{(2k+1)!}\int_0^1 y^{2k-1+\frac{1}{n}}dy=n+\sum_{k=1}^{+\infty}\frac{(-1)^k}{(2k+1)!}\frac{1}{2k+\frac{1}{n}}\]
最终我们得到
\[\int_0^1\frac{\sin x^n}{x^n}dx=1+\frac{1}{n}\sum_{k=1}^{+\infty}\frac{(-1)^k}{(2k+1)!}\frac{1}{2k+\frac{1}{n}}\]
取对数,要证明的表达式化为
\[\lim_{n\to+\infty}n\ln \left(1+\frac{1}{n}\sum_{k=1}^{+\infty}\frac{(-1)^k}{(2k+1)!}\frac{1}{2k+\frac{1}{n}}\right)=\sum_{k=1}^{+\infty}\left(\frac{(-1)^k}{2k(2k+1)!}\right)\]
\[\left|\sum_{k=1}^{+\infty}\frac{(-1)^k}{(2k+1)!}\frac{1}{2k+\frac{1}{n}}\right|\le \sum_{k=1}^{+\infty}\frac{1}{(2k+1)!}<+\infty\]
\[\lim_{n\to+\infty}\frac{1}{n}\sum_{k=1}^{+\infty}\frac{(-1)^k}{(2k+1)!}\frac{1}{2k+\frac{1}{n}}=0\]
\[\lim_{n\to+\infty}n\ln \left(1+\frac{1}{n}\sum_{k=1}^{+\infty}\frac{(-1)^k}{(2k+1)!}\frac{1}{2k+\frac{1}{n}}\right)=\lim_{n\to+\infty}\sum_{k=1}^{+\infty}\frac{(-1)^k}{(2k+1)!}\frac{1}{2k+\frac{1}{n}}\]
现在我们希望极限与级数和交换顺序,考虑$[0,1]$上的函数项级数
\[f(x)=\sum_{k=1}^{+\infty}\frac{(-1)^k}{(2k+1)!}\frac{1}{2k+x}\]
仍然由优级数判别法一致收敛,故由$\mathrm{Heine}$定理原式
\[=\lim_{n\to +\infty}f(\frac{1}{n})=f(0)=\sum_{k=1}^{+\infty}\frac{(-1)^k}{(2k+1)!}\frac{1}{2k}\]
作者: TangSong    时间: 昨天 02:00
 
试题(1c):证明
 
\[\lim_{n\to +\infty}\frac{1}{n}\sum_{k=1}^n\ln(1+\frac{k^2-k}{n^2})=\ln 2-2+\frac{\pi}{2}\]
注意到下面式子中第一项是一个$\mathrm{Riemann}$和我们有
\[\lim_{n\to +\infty}\frac{1}{n}\sum_{k=1}^n\ln(1+\frac{k^2}{n^2})=\int_0^1\ln(1+x^2)dx=x\ln(1+x^2)\left|_0^1\right.-\int_0^1\frac{2x^2}{1+x^2}dx=\ln 2-2+\frac{\pi}{2}\]
我们只需要再证明
\[\lim_{n\to +\infty}\frac{1}{n}\sum_{k=1}^n\ln(1+\frac{k^2-k}{n^2})=\lim_{n\to +\infty}\frac{1}{n}\sum_{k=1}^n\ln(1+\frac{k^2}{n^2})\]
利用不等式
\[x,y\ge 0,\left|\ln(1+x)-\ln(1+y)\right|=\left|\frac{x-y}{1+\xi}\right|\le|x-y|\]
\[\left|\frac{1}{n}\sum_{k=1}^n\ln(1+\frac{k^2-k}{n^2})-\frac{1}{n}\sum_{k=1}^n\ln(1+\frac{k^2}{n^2})\right|\le \frac{1}{n}\sum_{k=1}^n\left|\ln(1+\frac{k^2-k}{n^2})-\ln(1+\frac{k^2}{n^2})\right|\]
\[\le \frac{1}{n}\sum_{k=1}^n\frac{k}{n^2}=\frac{n+1}{2n^2}\to 0\]
作者: TangSong    时间: 昨天 02:54
试题(8)
$f(x)$在$[1,+\infty)$上二次可导,$\forall x\in [1,+\infty),f(x)>0,f''(x)\le 0,f(+\infty)=+\infty$
证明
\[\lim_{s\to 0^+}\sum_{n=1}^{+\infty}\frac{(-1)^n}{f^s(n)}\]
存在并求之.
 
由二阶导数非正,$f'(x)$在$[1,+\infty)$单减,容易看出$f'$恒正.事实上若有某个$x_0,f'(x_0)\le 0$则由单调性
\[\forall x\ge x_0,f'(x)\le f'(x_0)\le 0,f(x)\le f(x_0)\]与$f(+\infty)=+\infty$矛盾.因此$f$在$[1,+\infty)$严增.
我们将收敛性的证明与求值放在一起进行.
\[S_{2n}(s)=\sum_{k=1}^n \left(\frac{1}{f^s(2k)}-\frac{1}{f^s(2k-1)}\right)\]
注意和式中每个括号都是负的且级数通项趋于$0$,只需要证明对固定的$s>0,S_{2n}(s)$有下界则
\[\lim_{n\to +\infty}S_{2n}(s)\]存在且等于
\[\sum_{n=1}^{+\infty}\frac{(-1)^n}{f^s(n)}\]
由$\mathrm{Lagrange}$中值定理,
\[\frac{1}{f^s(2k)}-\frac{1}{f^s(2k-1)}=\frac{-sf'(\xi)}{f^{s+1}(\xi)},\xi\in (2k-1,2k)\]
注意$f$单增而$f'$单减我们有
\[\frac{-sf'(2k-1)}{f^{s+1}(2k-1)}\le\frac{1}{f^s(2k)}-\frac{1}{f^s(2k-1)}\le \frac{-sf'(2k)}{f^{s+1}(2k)}\]
\[\sum_{k=1}^n\frac{-sf'(2k-1)}{f^{s+1}(2k-1)}\le S_{2n}(s)\le \sum_{k=1}^n\frac{-sf'(2k)}{f^{s+1}(2k)}\]
利用面积原理的思想来估计左右两端.
由单调性$k\ge 2$时
\[\frac{f'(2k-1)}{f^{s+1}(2k-1)}\le \frac{1}{2}\int_{2k-3}^{2k-1} \frac{f'(t)}{f^{s+1}(t)}dt\]
\[\sum_{k=2}^{+\infty}\frac{f'(2k-1)}{f^{s+1}(2k-1)}\le\frac{1}{2}\int_{1}^{+\infty} \frac{f'(t)}{f^{s+1}(t)}dt=\frac{1}{2}\frac{-1}{sf^s(t)}\left|_{t=1}^{t=+\infty}\right.=\frac{1}{2sf^s(1)}\]
 
$S_{2n}(s)$有下界故极限存在.再次利用面积原理
$k\ge 1$时
\[\frac{f'(2k)}{f^{s+1}(2k)}\ge \frac{1}{2}\int_{2k}^{2k+2} \frac{f'(t)}{f^{s+1}(t)}dt\]
\[\sum_{k=1}^{+\infty}\frac{f'(2k)}{f^{s+1}(2k)}\ge\frac{1}{2}\int_{2}^{+\infty} \frac{f'(t)}{f^{s+1}(t)}dt=\frac{1}{2}\frac{-1}{sf^s(t)}\left|_{t=2}^{t=+\infty}\right.=\frac{1}{2sf^s(2)}\]
 
\[-s\left(\frac{f'(1)}{f^{s+1}(1)}+\frac{1}{2sf^s(1)}\right)\le \lim_{n\to +\infty}S_{2n}(s)\le -s\frac{1}{2sf^s(2)}\]
由前面说明就有
\[-s\left(\frac{f'(1)}{f^{s+1}(1)}+\frac{1}{2sf^s(1)}\right)\le \sum_{n=1}^{+\infty}\frac{(-1)^n}{f^s(n)}\le -s\frac{1}{2sf^s(2)}\]
而上式左右两端在$s\to 0^+$时极限都是$-\frac{1}{2}$故
\[\lim_{s\to 0^+}\sum_{n=1}^{+\infty}\frac{(-1)^n}{f^s(n)}=-\frac{1}{2}\] 

中科院2018研究生入学考试 数学分析+高等代数
数学分析部分
 
01. (15pt) 计算极限
\[\lim_{x\to\infty}\left(\sin\frac1x+\cos\frac1x\right)^{x}\text{.}\]
02. (15pt) 计算极限
\[\lim_{x\to 0} \left(\frac{4+\mathrm{e}^{\frac1x}}{2+\mathrm{e}^{\frac4x}}+\frac{\sin x}{|x|} \right)\text{.}\]
03. (15pt) 判断 (并证明) 函数 $f(x,y)=\sqrt{|{xy}|}$ 在点 $(0,0)$ 处的可微性.
 
04. (15pt) 求三个实常数 $a,b,c$,使得下式成立
\[\lim_{x\to 0}\frac1{\tan x -ax}\int_b^x\frac{s^2}{\sqrt{1-s^2}}\,\mathrm{d}s =c\text{.}\]
05. (15pt) 计算不定积分
\[\int\frac{\mathrm{d}x}{\sin^6 x+\cos^6 x}\text{.}\]
06. (15pt) 设函数 $f(x)$ 在 $[-1,1]$ 上二次连续可微,$f(0)=0$,证明:
\[
\left|\int_{-1}^1 f(x)\,\mathrm{d}x\right|\leq\frac{M}{3},\quad \text{其中 }M=\max_{x\in[-1,1]}\left|f''(x)\right|\text{.}
\]
07. (15pt) 求曲线 $y=\dfrac12x^2$ 上的点,使得曲线在该点处的法线被曲线所截得的线段长度最短.
 
08. (15pt) 设 $x>0$,证明
\[\sqrt{x+1}-\sqrt{x}=\frac1{2\sqrt{x+\theta}}\text{,}\]其中 $\theta=\theta(x)>0$,并且 $\lim\limits_{x\to 0}\theta(x)=\dfrac 14$.
 
09. (15pt) 设
\[u_n(x)=\frac{(-1)^n}{(n^2-n+1)^x}\quad (n\geq 0)\text{,}\]求函数 $f(x)=\sum\limits_{n=0}^{\infty}u_n(x)$ 的绝对收敛、条件收敛以及发散的区域.
 
10. (15pt) 证明
\[\frac15<\int_0^1\frac{x\mathrm{e}^x}{\sqrt{x^2-x+25}}\,\mathrm{d}x<\frac{2\sqrt{11}}{33}\text{.}\]
 
高等代数部分
 
一、(20pt) 设 $p(x),q(x),r(x)$ 都是数域 $\mathbb{k}$ 上的正次数多项式,而且 $p(x)$ 与 $q(x)$ 互素,$\mathrm{deg}(r(x))<\mathrm{deg}(p(x))+\mathrm{deg}(q(x))$.证明:存在数域 $\mathbb{k}$ 上的多项式 $u(x),v(x)$,满足 $\mathrm{deg}(u(x))<\mathrm{deg}(p(x)),\,\mathrm{deg}(v(x))<\mathrm{deg}(q(x))$,使得
\[\frac{r(x)}{p(x)q(x)}=\frac{u(x)}{p(x)}+\frac{v(x)}{q(x)}\text{.}\]
二、(20pt) 设 $n$ 阶方阵 $M_n=\left(|i-j|\right)_{1\leq i,j \leq n}$,令 $D_n=\mathrm{det}(M_n)$ ($M_n$ 的行列式).
  (1) 计算 $D_4$;
  (2) 证明 $D_n$ 满足递推关系式 $D_n=-4D_{n-1}-4D_{n-2}$;
  (3) 求 $n$ 阶方阵 $A_n=\left(\left|\frac1i-\frac1j\right|^{\llap{\phantom{b}}}\right)_{1\leq i,j \leq n}$ 的行列式 $\mathrm{det}(A_n)$.
 
三、(20pt) 设 $A,B$ 均是 $n$ 阶方阵,满足 $AB=0$.证明
  (1) $\mathrm{rank}(A)+\mathrm{rank}(B) \leq n$;
  (2) 对于方阵 $A$ 和正整数 $k\,(\mathrm{rank}(A) \leq k \leq n)$,必存在方阵 $B$,使得
\[\mathrm{rank}(A)+\mathrm{rank}(B)=k\text{.}\]
四、(20pt) 通过正交变换将下面的实二次型化成标准型:
\[q(x_1,x_2,x_3)=5x_1^2+5x_2^2+5x_3^2-2x_1x_2-2x_2x_3-2x_1x_3\text{.}\]
五、(20pt) 设 $A$ 和 $B$ 是两个 $n$ 阶实矩阵,并且 $A$ 是对称正定矩阵,$B$ 是反对称矩阵.证明:$A+B$ 是可逆矩阵.
 
六、(20pt) 设 $A$ 是 $n$ 阶复数矩阵,且 $A=\left(\begin{array}{l} A_1\\ A_2\end{array}\right)$,令
\[V_1=\left\{\,x\in\mathbb{C}^n\,\middle|\,A_1 x=0\,\right\},\quad V_1=\left\{\,x\in\mathbb{C}^n\,\middle|\,A_2 x=0\,\right\}\text{,}
\]证明:矩阵 $A$ 可逆的充分必要条件是向量空间 $\mathbb{C}^n$ 能够表示成子空间 $V_1$ 与 $V_2$ 的直和:$\mathbb{C}^n=V_1 \oplus V_2$.
 
七、(15pt) 证明:$8$ 个满足 $A^3=0$ 的 $5$ 阶复数矩阵中必有两个相似.
 
八、(15pt) $\mathbb{R}$ 上所有 $n\,(n\geq 2)$ 阶方阵构成的线性空间 $V=\mathbb{R}^{n \times n}$ 上的线性变换 $f:\, V \to V$ 定义为
\[f(A)=A+A'\quad \forall A\in V\text{,}\]其中 $A'$ 为 $A$ 的转置.求 $f$ 的特征值、特征子空间、极小多项式.

第九题的解答
 
 
9.  设 $B_R=\{(x,y): x^2+y^2< R^2\},u\in C^2( B_R)\cap C(\overline {B_R})$ .
 
1) 若$\Delta u\geqslant 0$, 证明
\[\max_{(x,y)\in\overline {B_R}} u(x,y)= \max_{(x,y) \in \partial B_R} u(x,y).\]
 
证明 对任意$\varepsilon>0$, 令$v_\varepsilon
(x,y)=u(x,y)+\varepsilon (x^2+y^2)$, 则
\[\Delta v_\varepsilon (x,y)=\Delta u(x,y)+4\varepsilon\geqslant 4\varepsilon.\]
由此用反证法易证
\[\max_{(x,y)\in\overline {B_R}} v_\varepsilon (x,y)= \max_{(x,y) \in \partial B_R} v_\varepsilon(x,y).\]
令$\varepsilon\to 0^+$, 即得
\[\max_{(x,y)\in\overline {B_R}} u(x,y)= \max_{(x,y) \in \partial B_R} u(x,y).\]
 
 
2).  若$\Delta u(x,y)=0$, 则
\[\frac{d}{dr}\left(\frac{1}{2\pi r}\int_{\partial B_r}u(x,y)ds\right)=0, 0\leqslant r\leqslant R.\]
 
 
证  注意到
\[\frac{1}{2\pi r}\int_{\partial B_r}u(x,y)ds=\frac{1}{2\pi}\int_0^{2\pi}u(r\cos\theta,r\sin\theta)d\theta=
\int_{\partial B_1}u(rx,ry)ds.\]从而根据Gauss公式, 得到
 
\begin{align*}\frac{d}{dr}\left(\frac{1}{2\pi r}\int_{\partial
B_r}u(x,y)ds\right)&=\frac{1}{2\pi}\int_{\partial B_1}(u_x(rx,ry)x+u_y(rx,ry)y)ds\\
&=\frac{1}{2\pi}\int_{\partial B_1} \frac{\partial
u(rx,ry)}{\partial
\nu}ds\\
&=\frac{1}{2\pi}\iint\limits_{\overline B_1}\Delta
u(rx,ry)dxdy\\
&=0.\end{align*}
3).  证明 若$\Delta u(x,y)=0$, 则
\[u(0,0)=\frac{1}{2\pi r}\int_{\partial B_r}u(x,y)ds.\]
 
证 根据2), 得到
\[\frac{1}{2\pi r}\int_{\partial B_r}u(x,y)ds=\lim_{r\to 0^+}\frac{1}{2\pi r}\int_{\partial B_r}u(x,y)ds=u(0,0).\]

2017-2018学年北京大学高等代数实验班期末试题2018.1.9
2018.1.9 上午8:30--10:30\\

安金鹏

据悉今年使用的教材是 K. Hoffman, R. Kunze: Linear Algebra

反响好我再发出期中试卷

一、设矩阵$A\in \mathbb{R}^{4\times 4}$的矩阵元均为$1$或$-1$, 求$\det A$的最大值.

二、设$V$是所有从有限域$F_p$到自身的映射构成的$F_p$-线性空间. 定义$T, U\in L(V)$为
$$ T(f)(t)=f(-t), \ \  U(f)(t)=f(t+1)-f(t), \ \ \forall \ f\in V, t\in F_p.$$
求$\det T$和$\det U$.

三、设$V$是有限维$F$-空间, $W$是$V$的子空间, $T\in L(V)$满足$T(W)\subset W$. 定义$T_W\in L(W)$和$T_{V/W}\in L(V/W)$为
$$T_W(\alpha)=T(\alpha), \alpha\in W,$$
$$T_{V/W}(\alpha+W)=T(\alpha)+W, \alpha\in V.$$
证明$\det T=\det T_W \det T_{V/W}$.

四、设$A\in F^{n\times n}$, $V$和$W$是$F^n$的子空间. 证明下述等价:

(a) 对任意的$a\in V-\{0\}$, 存在$\beta\in W$使得$\alpha A\beta^t\neq 0$.

(b) 对任意的$\gamma\in F^n$, 存在$\beta\in W$使得对任意的$\alpha\in V$有$\alpha V\beta^t=\alpha\gamma^t$.

五、设$F$是无限域. 证明对多项式代数$F[x]$的任意有限维子空间$V$, 存在$F[x]$的理想$M$满足
$$V\cap M=\{0\}, \ \ \ V+M=F[x].$$

 


登录 *


loading captcha image...
(输入验证码)
or Ctrl+Enter